4

Recently I baked a spherical cake ($3$ cm radius) and invited over a few friends, $6$ of them, for dinner. When done with main course, I thought of serving this spherical cake and to avoid uninvited disagreements over the size of the shares, I took my egg slicer with parallel wedges(and designed to cut $6$ slices at a go; my slicer has $5$ wedges) and placed my spherical cake right in the exact middle of it before pressing uniformly upon the slicer.

What should the relative placement of my wedges of the slicer be like so that I am able to equally distribute the cake among my $6$ friends?The set-up of the egg slicer looks something like this : enter image description here

  • 1
    Have you tried using calculus to find the volume of the portion of the solid ball of radius $r$, centered at the origin, lying between the planes $x = a$ and $x = b$ (with $-r \leq a < b \leq r$)...? (Not claiming this is optimal; because you want _six_ pieces, there's an obvious specific case to consider.) – Andrew D. Hwang Sep 14 '16 at 10:33
  • I think I finally get this question. The slicer is made of 5 **parallel** blades! – Ian Miller Sep 14 '16 at 12:53
  • @IanMiller Yes indeed! (You can see a "cross-section" of the desired slicer in my answer :D) – Bobson Dugnutt Sep 14 '16 at 13:14
  • @ Ian, yes the slicer is made of 5 parallel blades and the aim is to cut 6 equal shares of the spherical cake. –  Sep 14 '16 at 13:15
  • 1
    But if they are parallel then they aren't equally spaced. And @Lovsovs answer shows they aren't equally spaced. – Ian Miller Sep 14 '16 at 13:20
  • 2
    @IanMiller See Naveen's [last question](http://math.stackexchange.com/questions/1926312/spherical-cake-and-the-egg-slicer) for a picture of the situation (but there with equal spacings that wouldn't produce equal shares of the cake - hence this question). – Bobson Dugnutt Sep 14 '16 at 13:26
  • @ Ian Miller : and what has equally spaced got to do with the equal share ? –  Sep 14 '16 at 13:28
  • 2
    Your question stated that you did the cut with your "egg slicer with equally spaced wedges" were as you actually wanted one with unequal spaces which made equal shares. – Ian Miller Sep 14 '16 at 14:30
  • Ah, thanks for pointing that out. I just did the edit. –  Sep 14 '16 at 15:08
  • 3
    Thanks. That threw me lots at first as I hadn't seen your other question. – Ian Miller Sep 14 '16 at 15:32

2 Answers2

7

Let $R$ be the radius of your spherical cake. We will consider the case when we need to divide the cake into $n$ equally sized pieces.

Each piece will then have volume $\frac{4}{3}\frac{\pi R^3}{n}.$

We can calculate the volume of each slice by setting up a series of integrals, where we integrate over an infinitesimally thin cylinder with volume $dV=\pi r^2 dy,$ where $r\leq R$ is the radius of the cylinder (like this, but where $r$ here is the radius at height $h=R-y$). We have (draw this for yourself to see it) $r^2+y^2=R^2,$ which gives

$$\frac{4}{3}\frac{\pi R^3}{n}=\pi\int_{a_i}^{b_i}(R^2-y^2)dy=\pi\left(R^2(b_i-a_i) +\frac{a_i^3-b_i^3}{3} \right),$$ where $a_i$ is the value of $y$ where the $i$th slice starts and $b_i$ is the value where it ends, with $a_1=-R$ and $b_n=R$. Note that we have $a_{i+1}=b_i.$

We thus have $n$ equations with $n-1$ unknowns:

\begin{align} \frac{4}{3}\frac{ R^3}{n}&=R^2(b_1-a_1) +\frac{a_1^3-b_1^3}{3}=R^2(b_1-(-R)) +\frac{(-R)^3-b_1^3}{3}\\ &=R^2(b_2-a_2) +\frac{a_2^3-b_2^3}{3}=R^2(b_2-b_1) +\frac{b_1^3-b_2^3}{3} \\ &\quad\quad \quad\;\;\;\;\quad \quad\quad\quad\;\quad\quad \quad\vdots \\ &= R^2(b_n-a_n) +\frac{a_n^3-b_n^3}{3}=R^2(R-b_{n-1}) +\frac{b_{n-1}^3-R^3}{3}. \end{align}

As this is non-linear, it probably most fruitful to solve it numerically.

For $n=6$, in units of cake-radii (so $R=1$, but we can always scale), we have

$$b_1\approx-0.4817, \quad b_2\approx-0.2261,$$

which is all we need because of the symmetry ($b_3=0$). So your cake/egg-slicer should have distance

$$\Delta b_{01}=0-b_1\approx 0.4817,\Delta b_{12}=b_2-b_1\approx 0.2556,\Delta b_{23}=0-b_2 \approx 0.2261.$$

So your egg-slicer should look a little something like this:

$\quad\quad\quad\;\;\;$enter image description here

Here is the same as above, but for $n\in[1,20]$:

enter image description here

I must say that I'm surprised how broad the outermost pieces should be in order for them to have a volume equal to the rest, but this is probably my brain not being able to grasp exponentiation (here taking the third power).

Here is the (probably very ugly, but working) Mathematica code:

ClearAll["Global`*"];
M = 20;
For[
 n = 1, n < M, n++,
 ans = b /. 
   FullSimplify[
    Assuming[Element[n, Integers], 
     Solve[4/n == 3 (b - a) + a^3 - b^3, b]]];
 B[a_] = N[Re[ans[[3]]]];
 tabs[n] = {RecurrenceTable[{h[k + 1] == B[h[k]], h[1] == -1}, 
    h, {k, 1, n + 1}]};
 ]
NumberLinePlot[Table[tabs[k], {k, 1, n}], {x, -1, 1}]

As shown in this answer, we can find the position of the first blade to be (in the case of $R=1$)

$$ x(n)=2\cos \left( {\frac{{\alpha - 2\pi }} {3}} \right), \quad\quad \text{with } {\,\alpha = \arctan \left( {\frac{{2\sqrt {\left( {n - 1} \right)} }} {{n - 2}}} \right)}, \text{ for } n>2.$$

But feeding this answer into the next cubic equation and trying to solve just seems masochistic, hence the above call for numerical methods. Note that as $\tan (0)=0$ we have $\lim_{n\rightarrow \infty}x(n)=-1$ as expected.

Bobson Dugnutt
  • 10,499
  • 3
  • 24
  • 43
  • an extra like for that "line diagram" at the end of the answer. –  Sep 14 '16 at 13:16
  • 2
    @naveendankal Then you'll love my latest edit :P – Bobson Dugnutt Sep 14 '16 at 20:12
  • I surely do . +1 for that :D –  Sep 15 '16 at 00:56
  • 2
    Loving those plots! – dafinguzman Sep 15 '16 at 15:21
  • 2
    @Lovsovs, following our exchange of ideas on the other related post, I was curious to see if the *depressed cubic* has "viable" solutions also for the position of the different slices. Interestingly enough, the structure of the solutions remains also when introducing the $k$ parameter (position of other blades): I deem it might be of general interest, so I posted below. – G Cab Sep 16 '16 at 22:47
  • 2
    @GCab Very nice (+1) - I didn't expect that the other cuts could be found analytically (in a closed form) at all, well done! – Bobson Dugnutt Sep 16 '16 at 22:54
  • 2
    @Lovsovs, indeed before working on the other post, I would have not have spent even a minute to try and find it! definitely cubic equations have some interesting aspects to be discovered, and the method taken from A.Cauli is very helpful the clear the fog around them. – G Cab Sep 16 '16 at 23:20
  • 1
    @Lovsovs, and, by the way, I was not expecting that it was easier to cut the emi-sphere than the whole (!?). – G Cab Sep 16 '16 at 23:22
  • 1
    @GCab You mean "hemisphere", right? I guess it is natural to work with one half only, because of the symmetry. – Bobson Dugnutt Sep 16 '16 at 23:50
  • @Lovsovs, ahi .. the h's in English .. – G Cab Sep 17 '16 at 13:29
4

Let me recast Lovsovs' answer in another perspective.
Consider at first to split an hemi-sphere into $n$ parts of equal volume. Let designate as $h(k,n)$ the relative position of the $k$-th cut along the radius of the emisphere starting from the base circle (corresponding to $k=0$). $$ 0 \leqslant h(k,n) \leqslant 1\quad \left| {\;0 \leqslant k \leqslant n} \right. $$ Thus we shall have $$ \frac{2} {3}\pi R^{\,3} \frac{k} {n} = \pi \int_{y\; = \;0}^{h\,R} {\left( {R^{\,2} - y^{\,2} } \right)dy} = \pi \left( {R^{\,2} R\,h - \frac{1} {3}R^{\,3} h^{\,3} } \right) $$ that means $$ h^{\,3} - 3h + 2\frac{k}{n} = 0 $$ We will proceed and solve such depressed cubic equation according to the method indicated in this work by Alessandra Cauli, refer also to the answer to this post.
Putting apart the case $k=0$, for $\;1 \leqslant k \leqslant n$ we define $$ u = \sqrt[{3\,}]{{ - \frac{q} {2} + \sqrt {\frac{{q^{\,2} }} {4} + \frac{{p^{\,3} }} {{27}}} }} $$ where $p$ and $q$ are respectively the coefficients of $h^1$ and $h^0$. The 2nd radical is $$ \frac{{q^{\,2} }} {4} + \frac{{p^{\,3} }} {{27}} = \left( {\frac{k} {n}} \right)^{\,2} - 1 = - \frac{{n^{\,2} - k^{\,2} }} {{n^{\,2} }} \leqslant 0 $$ which being non-positive tells us that there are three real solutions.
Completing the calculation for $u$ $$ \begin{gathered} u = \sqrt[{3\,}]{{ - \frac{k} {n} + i\frac{1} {n}\sqrt {n^{\,2} - k^{\,2} } }} = \frac{1} {{\sqrt[{3\,}]{n}}}\sqrt[{3\,}]{{n\,e^{\,i\,\alpha } }} = \hfill \\ = e^{\,\,i\,\alpha \,/\,3} \quad \left| \begin{gathered} \;1 \leqslant k \leqslant n \hfill \\ \;\alpha = \arctan _{\text{4Q}} \left( { - k,\sqrt {n^{\,2} - k^{\,2} } } \right) = \pi - \arctan \left( {\sqrt {\left( {\frac{n} {k}} \right)^{\,2} - 1} } \right) = \hfill \\ \;\;\;\; = \pi - \beta \hfill \\ \end{gathered} \right. \hfill \\ \end{gathered} $$ We take then $$ v = - \frac{p} {{3\,u}} = \frac{1} {u}\quad \quad \omega = e^{\,i\,\frac{{2\pi }} {3}} = 1/2\left( { - 1 + i\sqrt 3 } \right) $$ and arrive to obtain the three solutions as $$ \begin{gathered} 1 \leqslant k \leqslant n\quad 0 \leqslant \beta = \arctan \left( {\sqrt {\left( {\frac{n} {k}} \right)^{\,2} - 1} } \right) < \frac{\pi } {2} \hfill \\ \left\{ \begin{gathered} h_{\,1} = e^{\,i\,\alpha /3} + e^{\, - \,i\,\alpha /3} = 2\cos \left( {\frac{\pi } {3} - \frac{\beta } {3}} \right) \hfill \\ h_{\,2} = e^{\,i\,\alpha /3 + 2\pi /3} + e^{\, - \,i\,\alpha /3 - 2\pi /3} = - 2\cos \left( {\frac{\beta } {3}} \right) \hfill \\ h_{\,3} = e^{\,i\,\alpha /3 - 2\pi /3} + e^{\, - \,i\,\alpha /3 + 2\pi /3} = 2\cos \left( {\frac{\pi } {3} + \frac{\beta } {3}} \right) \hfill \\ \end{gathered} \right. \hfill \\ \end{gathered} $$ of which we can easily determine that only $h_3$ respect the physical conditions of our problem.

Passing now to split the entire sphere, imagine it consisting of the two halves, placed base-to-base, one on the positive and one on the negative $h$ axis.
So, when $n$ is even we will place the cuts at $k = 0,\; \pm 2, \cdots ,\; \pm \left( {n - 2} \right)$,
while for $n$ odd at $k = \pm 1,\; \pm 3, \cdots ,\; \pm \left( {n - 2} \right)$.
In conclusion, always as a ratio to the radius, the cutting blades shall be positioned at: $$ \left\{ \begin{gathered} 2 \leqslant n\quad 0 \leqslant k \leqslant \left\lfloor {\frac{n} {2}} \right\rfloor - 1 \hfill \\ \beta \left( {k,n} \right) = \arctan \left( {\sqrt {\left( {\frac{n} {k}} \right)^{\,2} - 1} } \right) \hfill \\ h_{\,T} (k,n) = \pm 2\cos \left( {\frac{\pi } {3} + \frac{{\beta (2k + \bmod (n,2),\,n)}} {3}} \right) \hfill \\ \end{gathered} \right. $$ (if you can accept that$\pm 0=0$).

G Cab
  • 33,333
  • 3
  • 19
  • 60